Using induction to prove a sequence $a_1=1$ and $a_{n+1}=3-frac{1}{a_n}$ is increasing












1












$begingroup$


I have a sequence defined by $a_1=1$ and $a_{n+1}=3-frac{1}{a_n}$. I need to use induction to show that the sequence is increasing and $a_n<3$ for all $n$.



Also to deduce that $a_n$ is convergence and find its limit.



So far I have found: $a_n<a_{n+1}<3$ and $a_{n+1}>0$



Then $a_{n+1}=3-frac{1}{a_n}$ with $a_{n+2}=3-frac{a_n}{3a_n-1}$



But not really sure how to go any further even if this is the right way to go in the first place? Any input would be greatly be appreciated!










share|cite|improve this question











$endgroup$

















    1












    $begingroup$


    I have a sequence defined by $a_1=1$ and $a_{n+1}=3-frac{1}{a_n}$. I need to use induction to show that the sequence is increasing and $a_n<3$ for all $n$.



    Also to deduce that $a_n$ is convergence and find its limit.



    So far I have found: $a_n<a_{n+1}<3$ and $a_{n+1}>0$



    Then $a_{n+1}=3-frac{1}{a_n}$ with $a_{n+2}=3-frac{a_n}{3a_n-1}$



    But not really sure how to go any further even if this is the right way to go in the first place? Any input would be greatly be appreciated!










    share|cite|improve this question











    $endgroup$















      1












      1








      1





      $begingroup$


      I have a sequence defined by $a_1=1$ and $a_{n+1}=3-frac{1}{a_n}$. I need to use induction to show that the sequence is increasing and $a_n<3$ for all $n$.



      Also to deduce that $a_n$ is convergence and find its limit.



      So far I have found: $a_n<a_{n+1}<3$ and $a_{n+1}>0$



      Then $a_{n+1}=3-frac{1}{a_n}$ with $a_{n+2}=3-frac{a_n}{3a_n-1}$



      But not really sure how to go any further even if this is the right way to go in the first place? Any input would be greatly be appreciated!










      share|cite|improve this question











      $endgroup$




      I have a sequence defined by $a_1=1$ and $a_{n+1}=3-frac{1}{a_n}$. I need to use induction to show that the sequence is increasing and $a_n<3$ for all $n$.



      Also to deduce that $a_n$ is convergence and find its limit.



      So far I have found: $a_n<a_{n+1}<3$ and $a_{n+1}>0$



      Then $a_{n+1}=3-frac{1}{a_n}$ with $a_{n+2}=3-frac{a_n}{3a_n-1}$



      But not really sure how to go any further even if this is the right way to go in the first place? Any input would be greatly be appreciated!







      sequences-and-series limits inequality convergence induction






      share|cite|improve this question















      share|cite|improve this question













      share|cite|improve this question




      share|cite|improve this question








      edited Apr 20 at 12:35









      YuiTo Cheng

      2,82141139




      2,82141139










      asked Apr 20 at 11:01









      James odareJames odare

      40114




      40114






















          2 Answers
          2






          active

          oldest

          votes


















          1












          $begingroup$

          You have already done most of the work. By showing that $1 leq a_n < a_{n+1} < 3$ where $n in mathbb{Z}_+$, you have demonstrated that the sequence is increasing and bounded above by $3$. Now the Monotone Convergence Theorem says that any increasing sequence that is bounded above converges; indeed it converges to the supremum of ${a_n}_{n in mathbb{Z}_+}$. So you now know that $limlimits_{n to infty} a_n = alpha in mathbb{R}$ for some $$boxed{1 leq alpha leq 3}$$



          To find $alpha$, note that $f(x) = 3 - frac{1}{x}$ is a continuous real valued function for real $x > 0$. So, by continuity, and the fact that $alpha > 0$, we can put limits inside the function like so: $$limlimits_{n to infty}f(a_n) = f(limlimits_{n to infty}a_n)$$ But the LHS is just $limlimits_{n to infty}f(a_n) = limlimits_{n to infty}(3 - frac{1}{a_n}) = limlimits_{n to infty} a_{n + 1} = alpha$ while the RHS is just $f(limlimits_{n to infty}a_n) = f(alpha) = 3 - frac{1}{alpha}$. So, putting LHS $=$ RHS we get
          $$alpha = 3 - frac{1}{alpha}$$
          $$boxed{alpha^2 - 3 alpha + 1 = 0}$$ Now, solve this quadratic equation to get the value of $alpha$. You will get two values but only one of them is the answer. See if you can eliminate the wrong one using the properties of $alpha$.






          share|cite|improve this answer











          $endgroup$













          • $begingroup$
            Thank you very much!
            $endgroup$
            – James odare
            Apr 20 at 11:54










          • $begingroup$
            No problem, you're welcome.
            $endgroup$
            – ZeroXLR
            Apr 20 at 11:56



















          4












          $begingroup$

          Let $P(n)$ be the assertion “$a_ninleft[1,frac{3+sqrt5}2right)$ and $a_{n+1}>a_n$”.



          If $n=1$, then this is true, since $a_1=1$ and $a_2=2>1=a_1$.



          Let $ninmathbb N$ and assume that $P(n)$ holds. Then:





          • $a_{n+1}=3-frac1{a_n}<3$. since $a_ngeqslant1>0$;


          • $a_{n+1}geqslant1$, since $a_{n}geqslant1$ and so $3-frac1{a_n}geqslant3-frac11=2$.


          • $a_{n+2}-a_{n+1}=3-dfrac1{a_{n+1}}-a_{n+1}=dfrac{3a_{n+1}-1-{a_{n+1}}^2}{{a_{n+1}}^2}=dfrac{varphi(a_{n+1})}{{a_{n+1}}^2}$, where $varphi(x)=-x^2+3x-1$. But the roots of the quadratic polynomial $varphi(x)$ are $dfrac{3pmsqrt5}2$ and so, since $a_{n+1}$ is between them and the coefficient of $x^2$ in $varphi(x)$ is negative, $varphi(a_{n+1})>0$.


          So, $P(n+1)$ is proved.



          What I wrote above is an inductive proof of the fact that we always have $P(n)$. And it is very easy now to conclude that $lim_{ntoinfty}a_n=dfrac{3+sqrt5}2$.






          share|cite|improve this answer









          $endgroup$













          • $begingroup$
            Great explanation thank you
            $endgroup$
            – James odare
            Apr 20 at 11:54












          Your Answer








          StackExchange.ready(function() {
          var channelOptions = {
          tags: "".split(" "),
          id: "69"
          };
          initTagRenderer("".split(" "), "".split(" "), channelOptions);

          StackExchange.using("externalEditor", function() {
          // Have to fire editor after snippets, if snippets enabled
          if (StackExchange.settings.snippets.snippetsEnabled) {
          StackExchange.using("snippets", function() {
          createEditor();
          });
          }
          else {
          createEditor();
          }
          });

          function createEditor() {
          StackExchange.prepareEditor({
          heartbeatType: 'answer',
          autoActivateHeartbeat: false,
          convertImagesToLinks: true,
          noModals: true,
          showLowRepImageUploadWarning: true,
          reputationToPostImages: 10,
          bindNavPrevention: true,
          postfix: "",
          imageUploader: {
          brandingHtml: "Powered by u003ca class="icon-imgur-white" href="https://imgur.com/"u003eu003c/au003e",
          contentPolicyHtml: "User contributions licensed under u003ca href="https://creativecommons.org/licenses/by-sa/3.0/"u003ecc by-sa 3.0 with attribution requiredu003c/au003e u003ca href="https://stackoverflow.com/legal/content-policy"u003e(content policy)u003c/au003e",
          allowUrls: true
          },
          noCode: true, onDemand: true,
          discardSelector: ".discard-answer"
          ,immediatelyShowMarkdownHelp:true
          });


          }
          });














          draft saved

          draft discarded


















          StackExchange.ready(
          function () {
          StackExchange.openid.initPostLogin('.new-post-login', 'https%3a%2f%2fmath.stackexchange.com%2fquestions%2f3194560%2fusing-induction-to-prove-a-sequence-a-1-1-and-a-n1-3-frac1a-n-is-inc%23new-answer', 'question_page');
          }
          );

          Post as a guest















          Required, but never shown

























          2 Answers
          2






          active

          oldest

          votes








          2 Answers
          2






          active

          oldest

          votes









          active

          oldest

          votes






          active

          oldest

          votes









          1












          $begingroup$

          You have already done most of the work. By showing that $1 leq a_n < a_{n+1} < 3$ where $n in mathbb{Z}_+$, you have demonstrated that the sequence is increasing and bounded above by $3$. Now the Monotone Convergence Theorem says that any increasing sequence that is bounded above converges; indeed it converges to the supremum of ${a_n}_{n in mathbb{Z}_+}$. So you now know that $limlimits_{n to infty} a_n = alpha in mathbb{R}$ for some $$boxed{1 leq alpha leq 3}$$



          To find $alpha$, note that $f(x) = 3 - frac{1}{x}$ is a continuous real valued function for real $x > 0$. So, by continuity, and the fact that $alpha > 0$, we can put limits inside the function like so: $$limlimits_{n to infty}f(a_n) = f(limlimits_{n to infty}a_n)$$ But the LHS is just $limlimits_{n to infty}f(a_n) = limlimits_{n to infty}(3 - frac{1}{a_n}) = limlimits_{n to infty} a_{n + 1} = alpha$ while the RHS is just $f(limlimits_{n to infty}a_n) = f(alpha) = 3 - frac{1}{alpha}$. So, putting LHS $=$ RHS we get
          $$alpha = 3 - frac{1}{alpha}$$
          $$boxed{alpha^2 - 3 alpha + 1 = 0}$$ Now, solve this quadratic equation to get the value of $alpha$. You will get two values but only one of them is the answer. See if you can eliminate the wrong one using the properties of $alpha$.






          share|cite|improve this answer











          $endgroup$













          • $begingroup$
            Thank you very much!
            $endgroup$
            – James odare
            Apr 20 at 11:54










          • $begingroup$
            No problem, you're welcome.
            $endgroup$
            – ZeroXLR
            Apr 20 at 11:56
















          1












          $begingroup$

          You have already done most of the work. By showing that $1 leq a_n < a_{n+1} < 3$ where $n in mathbb{Z}_+$, you have demonstrated that the sequence is increasing and bounded above by $3$. Now the Monotone Convergence Theorem says that any increasing sequence that is bounded above converges; indeed it converges to the supremum of ${a_n}_{n in mathbb{Z}_+}$. So you now know that $limlimits_{n to infty} a_n = alpha in mathbb{R}$ for some $$boxed{1 leq alpha leq 3}$$



          To find $alpha$, note that $f(x) = 3 - frac{1}{x}$ is a continuous real valued function for real $x > 0$. So, by continuity, and the fact that $alpha > 0$, we can put limits inside the function like so: $$limlimits_{n to infty}f(a_n) = f(limlimits_{n to infty}a_n)$$ But the LHS is just $limlimits_{n to infty}f(a_n) = limlimits_{n to infty}(3 - frac{1}{a_n}) = limlimits_{n to infty} a_{n + 1} = alpha$ while the RHS is just $f(limlimits_{n to infty}a_n) = f(alpha) = 3 - frac{1}{alpha}$. So, putting LHS $=$ RHS we get
          $$alpha = 3 - frac{1}{alpha}$$
          $$boxed{alpha^2 - 3 alpha + 1 = 0}$$ Now, solve this quadratic equation to get the value of $alpha$. You will get two values but only one of them is the answer. See if you can eliminate the wrong one using the properties of $alpha$.






          share|cite|improve this answer











          $endgroup$













          • $begingroup$
            Thank you very much!
            $endgroup$
            – James odare
            Apr 20 at 11:54










          • $begingroup$
            No problem, you're welcome.
            $endgroup$
            – ZeroXLR
            Apr 20 at 11:56














          1












          1








          1





          $begingroup$

          You have already done most of the work. By showing that $1 leq a_n < a_{n+1} < 3$ where $n in mathbb{Z}_+$, you have demonstrated that the sequence is increasing and bounded above by $3$. Now the Monotone Convergence Theorem says that any increasing sequence that is bounded above converges; indeed it converges to the supremum of ${a_n}_{n in mathbb{Z}_+}$. So you now know that $limlimits_{n to infty} a_n = alpha in mathbb{R}$ for some $$boxed{1 leq alpha leq 3}$$



          To find $alpha$, note that $f(x) = 3 - frac{1}{x}$ is a continuous real valued function for real $x > 0$. So, by continuity, and the fact that $alpha > 0$, we can put limits inside the function like so: $$limlimits_{n to infty}f(a_n) = f(limlimits_{n to infty}a_n)$$ But the LHS is just $limlimits_{n to infty}f(a_n) = limlimits_{n to infty}(3 - frac{1}{a_n}) = limlimits_{n to infty} a_{n + 1} = alpha$ while the RHS is just $f(limlimits_{n to infty}a_n) = f(alpha) = 3 - frac{1}{alpha}$. So, putting LHS $=$ RHS we get
          $$alpha = 3 - frac{1}{alpha}$$
          $$boxed{alpha^2 - 3 alpha + 1 = 0}$$ Now, solve this quadratic equation to get the value of $alpha$. You will get two values but only one of them is the answer. See if you can eliminate the wrong one using the properties of $alpha$.






          share|cite|improve this answer











          $endgroup$



          You have already done most of the work. By showing that $1 leq a_n < a_{n+1} < 3$ where $n in mathbb{Z}_+$, you have demonstrated that the sequence is increasing and bounded above by $3$. Now the Monotone Convergence Theorem says that any increasing sequence that is bounded above converges; indeed it converges to the supremum of ${a_n}_{n in mathbb{Z}_+}$. So you now know that $limlimits_{n to infty} a_n = alpha in mathbb{R}$ for some $$boxed{1 leq alpha leq 3}$$



          To find $alpha$, note that $f(x) = 3 - frac{1}{x}$ is a continuous real valued function for real $x > 0$. So, by continuity, and the fact that $alpha > 0$, we can put limits inside the function like so: $$limlimits_{n to infty}f(a_n) = f(limlimits_{n to infty}a_n)$$ But the LHS is just $limlimits_{n to infty}f(a_n) = limlimits_{n to infty}(3 - frac{1}{a_n}) = limlimits_{n to infty} a_{n + 1} = alpha$ while the RHS is just $f(limlimits_{n to infty}a_n) = f(alpha) = 3 - frac{1}{alpha}$. So, putting LHS $=$ RHS we get
          $$alpha = 3 - frac{1}{alpha}$$
          $$boxed{alpha^2 - 3 alpha + 1 = 0}$$ Now, solve this quadratic equation to get the value of $alpha$. You will get two values but only one of them is the answer. See if you can eliminate the wrong one using the properties of $alpha$.







          share|cite|improve this answer














          share|cite|improve this answer



          share|cite|improve this answer








          edited Apr 20 at 12:28

























          answered Apr 20 at 11:29









          ZeroXLRZeroXLR

          1,821620




          1,821620












          • $begingroup$
            Thank you very much!
            $endgroup$
            – James odare
            Apr 20 at 11:54










          • $begingroup$
            No problem, you're welcome.
            $endgroup$
            – ZeroXLR
            Apr 20 at 11:56


















          • $begingroup$
            Thank you very much!
            $endgroup$
            – James odare
            Apr 20 at 11:54










          • $begingroup$
            No problem, you're welcome.
            $endgroup$
            – ZeroXLR
            Apr 20 at 11:56
















          $begingroup$
          Thank you very much!
          $endgroup$
          – James odare
          Apr 20 at 11:54




          $begingroup$
          Thank you very much!
          $endgroup$
          – James odare
          Apr 20 at 11:54












          $begingroup$
          No problem, you're welcome.
          $endgroup$
          – ZeroXLR
          Apr 20 at 11:56




          $begingroup$
          No problem, you're welcome.
          $endgroup$
          – ZeroXLR
          Apr 20 at 11:56











          4












          $begingroup$

          Let $P(n)$ be the assertion “$a_ninleft[1,frac{3+sqrt5}2right)$ and $a_{n+1}>a_n$”.



          If $n=1$, then this is true, since $a_1=1$ and $a_2=2>1=a_1$.



          Let $ninmathbb N$ and assume that $P(n)$ holds. Then:





          • $a_{n+1}=3-frac1{a_n}<3$. since $a_ngeqslant1>0$;


          • $a_{n+1}geqslant1$, since $a_{n}geqslant1$ and so $3-frac1{a_n}geqslant3-frac11=2$.


          • $a_{n+2}-a_{n+1}=3-dfrac1{a_{n+1}}-a_{n+1}=dfrac{3a_{n+1}-1-{a_{n+1}}^2}{{a_{n+1}}^2}=dfrac{varphi(a_{n+1})}{{a_{n+1}}^2}$, where $varphi(x)=-x^2+3x-1$. But the roots of the quadratic polynomial $varphi(x)$ are $dfrac{3pmsqrt5}2$ and so, since $a_{n+1}$ is between them and the coefficient of $x^2$ in $varphi(x)$ is negative, $varphi(a_{n+1})>0$.


          So, $P(n+1)$ is proved.



          What I wrote above is an inductive proof of the fact that we always have $P(n)$. And it is very easy now to conclude that $lim_{ntoinfty}a_n=dfrac{3+sqrt5}2$.






          share|cite|improve this answer









          $endgroup$













          • $begingroup$
            Great explanation thank you
            $endgroup$
            – James odare
            Apr 20 at 11:54
















          4












          $begingroup$

          Let $P(n)$ be the assertion “$a_ninleft[1,frac{3+sqrt5}2right)$ and $a_{n+1}>a_n$”.



          If $n=1$, then this is true, since $a_1=1$ and $a_2=2>1=a_1$.



          Let $ninmathbb N$ and assume that $P(n)$ holds. Then:





          • $a_{n+1}=3-frac1{a_n}<3$. since $a_ngeqslant1>0$;


          • $a_{n+1}geqslant1$, since $a_{n}geqslant1$ and so $3-frac1{a_n}geqslant3-frac11=2$.


          • $a_{n+2}-a_{n+1}=3-dfrac1{a_{n+1}}-a_{n+1}=dfrac{3a_{n+1}-1-{a_{n+1}}^2}{{a_{n+1}}^2}=dfrac{varphi(a_{n+1})}{{a_{n+1}}^2}$, where $varphi(x)=-x^2+3x-1$. But the roots of the quadratic polynomial $varphi(x)$ are $dfrac{3pmsqrt5}2$ and so, since $a_{n+1}$ is between them and the coefficient of $x^2$ in $varphi(x)$ is negative, $varphi(a_{n+1})>0$.


          So, $P(n+1)$ is proved.



          What I wrote above is an inductive proof of the fact that we always have $P(n)$. And it is very easy now to conclude that $lim_{ntoinfty}a_n=dfrac{3+sqrt5}2$.






          share|cite|improve this answer









          $endgroup$













          • $begingroup$
            Great explanation thank you
            $endgroup$
            – James odare
            Apr 20 at 11:54














          4












          4








          4





          $begingroup$

          Let $P(n)$ be the assertion “$a_ninleft[1,frac{3+sqrt5}2right)$ and $a_{n+1}>a_n$”.



          If $n=1$, then this is true, since $a_1=1$ and $a_2=2>1=a_1$.



          Let $ninmathbb N$ and assume that $P(n)$ holds. Then:





          • $a_{n+1}=3-frac1{a_n}<3$. since $a_ngeqslant1>0$;


          • $a_{n+1}geqslant1$, since $a_{n}geqslant1$ and so $3-frac1{a_n}geqslant3-frac11=2$.


          • $a_{n+2}-a_{n+1}=3-dfrac1{a_{n+1}}-a_{n+1}=dfrac{3a_{n+1}-1-{a_{n+1}}^2}{{a_{n+1}}^2}=dfrac{varphi(a_{n+1})}{{a_{n+1}}^2}$, where $varphi(x)=-x^2+3x-1$. But the roots of the quadratic polynomial $varphi(x)$ are $dfrac{3pmsqrt5}2$ and so, since $a_{n+1}$ is between them and the coefficient of $x^2$ in $varphi(x)$ is negative, $varphi(a_{n+1})>0$.


          So, $P(n+1)$ is proved.



          What I wrote above is an inductive proof of the fact that we always have $P(n)$. And it is very easy now to conclude that $lim_{ntoinfty}a_n=dfrac{3+sqrt5}2$.






          share|cite|improve this answer









          $endgroup$



          Let $P(n)$ be the assertion “$a_ninleft[1,frac{3+sqrt5}2right)$ and $a_{n+1}>a_n$”.



          If $n=1$, then this is true, since $a_1=1$ and $a_2=2>1=a_1$.



          Let $ninmathbb N$ and assume that $P(n)$ holds. Then:





          • $a_{n+1}=3-frac1{a_n}<3$. since $a_ngeqslant1>0$;


          • $a_{n+1}geqslant1$, since $a_{n}geqslant1$ and so $3-frac1{a_n}geqslant3-frac11=2$.


          • $a_{n+2}-a_{n+1}=3-dfrac1{a_{n+1}}-a_{n+1}=dfrac{3a_{n+1}-1-{a_{n+1}}^2}{{a_{n+1}}^2}=dfrac{varphi(a_{n+1})}{{a_{n+1}}^2}$, where $varphi(x)=-x^2+3x-1$. But the roots of the quadratic polynomial $varphi(x)$ are $dfrac{3pmsqrt5}2$ and so, since $a_{n+1}$ is between them and the coefficient of $x^2$ in $varphi(x)$ is negative, $varphi(a_{n+1})>0$.


          So, $P(n+1)$ is proved.



          What I wrote above is an inductive proof of the fact that we always have $P(n)$. And it is very easy now to conclude that $lim_{ntoinfty}a_n=dfrac{3+sqrt5}2$.







          share|cite|improve this answer












          share|cite|improve this answer



          share|cite|improve this answer










          answered Apr 20 at 11:33









          José Carlos SantosJosé Carlos Santos

          177k24138251




          177k24138251












          • $begingroup$
            Great explanation thank you
            $endgroup$
            – James odare
            Apr 20 at 11:54


















          • $begingroup$
            Great explanation thank you
            $endgroup$
            – James odare
            Apr 20 at 11:54
















          $begingroup$
          Great explanation thank you
          $endgroup$
          – James odare
          Apr 20 at 11:54




          $begingroup$
          Great explanation thank you
          $endgroup$
          – James odare
          Apr 20 at 11:54


















          draft saved

          draft discarded




















































          Thanks for contributing an answer to Mathematics Stack Exchange!


          • Please be sure to answer the question. Provide details and share your research!

          But avoid



          • Asking for help, clarification, or responding to other answers.

          • Making statements based on opinion; back them up with references or personal experience.


          Use MathJax to format equations. MathJax reference.


          To learn more, see our tips on writing great answers.




          draft saved


          draft discarded














          StackExchange.ready(
          function () {
          StackExchange.openid.initPostLogin('.new-post-login', 'https%3a%2f%2fmath.stackexchange.com%2fquestions%2f3194560%2fusing-induction-to-prove-a-sequence-a-1-1-and-a-n1-3-frac1a-n-is-inc%23new-answer', 'question_page');
          }
          );

          Post as a guest















          Required, but never shown





















































          Required, but never shown














          Required, but never shown












          Required, but never shown







          Required, but never shown

































          Required, but never shown














          Required, but never shown












          Required, but never shown







          Required, but never shown







          Popular posts from this blog

          How to put 3 figures in Latex with 2 figures side by side and 1 below these side by side images but in...

          In PowerPoint, is there a keyboard shortcut for bulleted / numbered list?

          IC on Digikey is 5x more expensive than board containing same IC on Alibaba: How? [on hold]